Does left-multiplication by compact operators turn strong-convergence into norm-convergence

compact-operatorsfunctional-analysisstrong-convergence

If $\{T_i\}_{i\in I}$ is a bounded net of operators on a Hilbert space $\mathscr H$, converging strongly to some operator $T$, and
if $K$ is a compact operator on $\mathscr H$, then the net
$\{T_iK\}_i$ is known to converge in norm to $TK$.

Question. Is it also true that $\{KT_i\}_i$ converges in norm to $KT$?


PS:

  1. The present question arouse in the comments following this answer and, while I can't remember ever questioning it, neither do I remember this being discussed anywhere. After a while I
    finally figured out the answer and I thought it would be nice to record it here.
  2. An affirmative answer to my question is implicitly assumed in the statement of this question.

Best Answer

This may come as a surprise but the answer is no. In fact it is no in a very strong way!!

There exists a sequence $\{T_n\}_n$ of bounded operators, strongly converging to zero, such that $\{KT_n\}_n$ does not converge to zero in norm for every nonzero bounded operator $K$, regardless of whether $K$ is compact or even finite rank!

Let $S:\ell ^2\to \ell ^2$ be the so called unilateral shift given by $$ S(x_0, x_1, x_2, x_3, \ldots ) = (0, x_0, x_1, x_2, \ldots ). $$ The adjoint of $S$ turns out to be given by $$ S^*(x_0, x_1, x_2, \ldots ) = (x_1, x_2, x_3, \ldots ), $$ from where one has that $S^*S$ is the identity operator.

Now let $T_n={S^*}^n$, so that $$ T_n(x_0, x_1, x_2, \ldots ) = (x_n, x_{n+1}, x_{n+2}, \ldots ), $$ and hence it is clear that $T_n\to 0$ strongly.

If $K$ is any bounded operator (compact or not) we have that $$ \|K\| = \|K{S^n}^*S^n\| \leq \|K{S^n}^*\|\|S^n\| = \|K{S^n}^*\| = \|KT_n\|, $$ so we can't have $\|KT_n\|\to 0$, unless $K=0$.


  1. As mentioned in the original post, if $K$ is compact, it is well known that "$T_n\to 0$ strongly" implies "$T_nK\to 0$ in norm".

  2. If $T_n^*\to 0$ strongly (e.g. if $T_n\to 0$ in the $^*$-strong topology, or if $T_n$ is self-adjoint and converges to zero strongly) then $T_n^*K^*\to 0$ in norm by (1) and hence $KT_n\to 0$ in norm as well.